Math Pro 數學補給站's Archiver

在遇到困難時要具備有三個自我的能力:
自我激勵、自我轉換、自我調節。

Joy091 發表於 2010-6-21 16:52

99松山家商

題目與答案

chiang 發表於 2010-6-22 07:35

可以請教一下
填充第3,6兩題
計算4,5,7
如何計算嗎?
謝謝

weiye 發表於 2010-6-22 13:43

填充第 3 題:

有一台測謊機,當受測者說實話時,它測得對方說實話的機率為0.9,而當受測者說謊話時,它測得對方說謊話的機率為 0.8。又有甲、乙、丙三人,他們不是說實話就是說謊話,每個人說實話的機率分別都是 0.5,而且不互相影響。今袋中有 2 白球3 黑球,從中隨機取出1 球,甲乙丙看完後都說是黑球,再接受此機器測謊,它卻宣稱甲乙說實話而丙說謊話。請問該球確實是黑球之機率為何?

解答:

\(\displaystyle P\left(抽到黑球\Big|\mbox{機器宣稱甲乙說實話而丙說謊話}\right)\)

\(\displaystyle =\frac{P(\mbox{抽到黑球且機器宣稱甲乙說實話而丙說謊話})}{P(\mbox{機器宣稱甲乙說實話而丙說謊話})}\)

\(\displaystyle =\frac{P(\mbox{抽到黑球且機器宣稱甲乙說實話而丙說謊話})}{P(\mbox{抽到黑球且機器宣稱甲乙說實話而丙說謊話})+P(\mbox{抽到白球且機器宣稱甲乙說實話而丙說謊話})}\)



\(\displaystyle =\frac{\displaystyle \frac{3}{5}\cdot\frac{9}{10}\cdot\frac{9}{10}\cdot\frac{1}{10}}{\displaystyle \frac{3}{5}\cdot\frac{9}{10}\cdot\frac{9}{10}\cdot\frac{1}{10}+\frac{2}{5}\cdot\frac{2}{10}\cdot\frac{2}{10}\cdot\frac{8}{10}}\)



\(\displaystyle =\frac{243}{307}.\)

weiye 發表於 2010-6-22 13:57

填充第 6 題

將 \(xy\) 平面上的區域 \(\displaystyle \left\{\begin{array}{ccc}x^2+y^2\leq1\\ x\leq0, y\geq0\end{array}\right.\) 繞 \(xy\) 平面上的直線 \(x = y\) 在空間中旋轉一圈所得的旋轉體體積為何?

解答:

將圖形順時針旋轉 \(45^\circ\),
[attach]244[/attach]

題目所求變成要求上圖區域繞 \(x\) 軸旋轉的體積。

所求體積 \(\displaystyle=2\int_0^{\frac{1}{\sqrt{2}}} \pi \left(\left(1-x^2\right)-x^2\right)dx=\frac{2\sqrt{2}\pi}{3}.\)

weiye 發表於 2010-6-22 14:32

演算題第 4 題:

[attach]245[/attach]

如圖,在座標平面上,將橢圓 \(\displaystyle\frac{x^2}{4}+\frac{y^2}{1}=1\) 的上半部及圓 \(x^2 + y^2 = 4\) 的下半部組合而成一封閉曲線(其中, \(A, B\) 為此橢圓與圓的交點)。

今有一光線從此橢圓的一焦點 \(F_1\) 射向橢圓上半部的 \(P\) 點,依光學原理反射至下半圓的 \(Q\) 點,再依光學原理反射至下半圓的 \(R\) 點,再依光學原理反射。

若 \(\overline{QR} //\overline{AB}\) 且在 \(R\) 點的反射線通過 \(F\) , 請求出四邊形 \(FPQR\) 的周長及面積。



解答:

如上圖,設 \(O\) 為原點,

顯然 \(F_1P+PF_2=\mbox{橢圓長軸長}=4.\)

在 \(Q,R\) 兩點的入射角等於反射角,所以 \(∠ 1=∠2\) 且 \(∠ 3=∠4\)

因為 \(F_1F_2//QR\),所以 \(∠ 2=∠ 5\) 且 \(∠ 3=∠7\)

因為 \(OQ=OR\),所以 \(∠ 2=∠3\)

因此,\(∠ 1=∠2=∠ 3=∠4=∠ 5=∠7\),

\(F_2Q=OF_2=\sqrt{3}\) 且 \(F_1R=OF_1=\sqrt{3}\)

因為 \(\triangle OQR\) 相似於 \(\triangle F_2QO\),

所以 \(\displaystyle QR:OR=OQ:OF_2\Rightarrow QR=2\times\frac{2}{\sqrt{3}}=\frac{4\sqrt{3}}{3}.\)

所以四邊形 \(FPQR\) 的周長 \(\displaystyle=4+\sqrt{3}+\sqrt{3}+\frac{4\sqrt{3}}{3}=4+\frac{10\sqrt{3}}{3}.\)





至於面積,沒想到更好的方法,只有想到硬作,

(就是硬找出 \(P\) 的坐標,再求面積。其實可以找每個點的坐標,剛剛的周長也不是問題。==)

在 \(\triangle OF_2Q\) 中,\(\displaystyle\cos∠OF_2Q=\frac{\left(\sqrt{3}\right)^2+\left(\sqrt{3}\right)^2-2^2}{2\cdot\sqrt{3}\cdot\sqrt{3}}=\frac{1}{3}.\)

所以,\(\displaystyle\cos∠OF_2P=\frac{-1}{3}\Rightarrow \tan∠OF_2P=-2\sqrt{2}\)

(順便多算一個 \(\displaystyle\sin∠OF_2P=\frac{2\sqrt{2}}{3}\))

可得直線 \(\overleftrightarrow{PQ}\) 的方程式為 \(y=-2\sqrt{2}\left(x+\sqrt{3}\right).\)

在與橢圓的方程式,解聯立方程式,可得 \(P\) 點的 \(y\) 坐標為 \(\displaystyle\frac{12\sqrt{2}-2\sqrt{6}}{33}.\)

所求面積\(=\triangle PF_1F_2\mbox{面積}+\triangle OF_2Q\mbox{面積}+\triangle OQR\mbox{面積}+\triangle ORF_1\mbox{面積}\)

\(\displaystyle= \frac{1}{2}\cdot2\sqrt{3}\cdot\frac{12\sqrt{2}-2\sqrt{6}}{33}+\frac{1}{2}\cdot\sqrt{3}\cdot\sqrt{3}\cdot\frac{2\sqrt{2}}{3}++\frac{1}{2}\cdot2\cdot2\cdot\frac{2\sqrt{2}}{3}++\frac{1}{2}\cdot\sqrt{3}\cdot\sqrt{3}\cdot\frac{2\sqrt{2}}{3}\)

\(\displaystyle=\frac{104\sqrt{2}+12\sqrt{6}}{33}.\)

(寫的很長,如有小錯希望不吝告知。感激。^__^)

weiye 發表於 2010-6-22 15:45

演算題第 7 題:

設 \(k\) 為實數。若對於任意實數 \(t\),關於四個未知實數 \(x, y, z,w\) 的方程組

\(\displaystyle\left\{\begin{array}{ccc} -tx+2y+0z+kw&=&0\\
-2x-ty+0z+0w&=&0\\
0x+4y+(-1-t)z-2w&=&0\\
0x+0y+2z+(-1-t)w&=&0\end{array}\right.\)

皆只有唯一一組解 \(x = 0, y = 0, z = 0,w = 0\),試求出 \(k\) 的範圍。

解答:

令 \(\displaystyle A=\left[\begin{array}{cccc}-t&2&0&k\\
-2&-t&0&0\\
0&4&(-1-t)&-2\\
0&0&2&(-1-t)
\end{array}\right]\)

因為 \(A\left[\begin{array}{c}x\\y\\z\\w\end{array}\right]=0\) 之 \(x,y,z,w\) 只有唯一一組解。

所以 \(A\) 的反矩陣存在 \(\Rightarrow det\left(A\right)\neq0\)

\(\Rightarrow t^4+2t^3+9t^2+8t+16k+20\neq0\)

令 \(f(t)=t^4+2t^3+9t^2+8t+16k+20\)

則對任意實數 \(t\),\(f(t)>0\) 恒成立。

由 \(\displaystyle f'(t)=0\Rightarrow t=\frac{-1}{2}\)

可知 \(f(t)\) 的最小值為 \(\displaystyle f(\frac{-1}{2})=\frac{256k+289}{16}\)

由 \(\displaystyle f(\frac{-1}{2})=\frac{256k+289}{16}>0\) 可得 \(\displaystyle k>\frac{-289}{256}.\)

故,當 \(\displaystyle k>\frac{-289}{256}\) 時,\(f(t)>0\) 恒成立 \(\Leftrightarrow A\) 的反矩陣成存在

\(\Leftrightarrow\)  題目的聯立方程組有唯一一組解。

weiye 發表於 2010-6-22 20:38

演算第 5 題:

(1)如下之街道圖中,每一小格皆是邊長 1 單位的正方形。
為了方便,我們簡稱此街道圖的規格為8×4。

[img]http://i.imgur.com/ECqHU.jpg[/img]

小松從 A 點出發,要沿著街道走捷徑到 B 點。因為溽暑難耐,
他每走 1 單位或每走2 單位就停下來休息一下。

請問,在如上所述的條件下,從 A 到 B 共有幾種走法?

(2)我們可將(1)中之街道圖規格改為 m×n (m,n為正整數),而在(1)所述的規
則下,考慮從 A 到B 共有幾種走法。

設規格為m×n , m×(n+1) , (m+1)×(n+1) 的街道圖所對應之走法數分別是
x , y , z ,請將 z 用 x, y 及 m,n 表示。





解答:

令 \(f(n) =\) 路徑長共需 \(n\) 單位時,每次走 \(1\) 或 \(2\) 單位就需停頓休息,所有停頓點排列的可能數。

則 \(f(1)=1,\, f(2)=2,\, f(n)=f(n-1)+f(n-2) \,\forall n\geq 3\)

[table=50%][tr][td] \(n\)[/td][td]1
[/td][td]2
[/td][td]3
[/td][td]4
[/td][td]5
[/td][td]6
[/td][td]7
[/td][td]8
[/td][td]9
[/td][td]10
[/td][td]11
[/td][td]12
[/td][/tr][tr][td] \(f(n)\)[/td][td] 1[/td][td]2
[/td][td]3
[/td][td]5
[/td][td]8
[/td][td]13
[/td][td] 21[/td][td] 34[/td][td] 55[/td][td] 89[/td][td]144
[/td][td]233
[/td][/tr][/table]




(1)

所求 \(=\mbox{A 到 B 的所有捷徑走法}\times f(12)=495\times233=115335.\)


(2)

\(\displaystyle x=\frac{\left(m+n\right)!}{m!n!}f(m+n)\Rightarrow f(m+n)=\frac{x\cdot m!\cdot n!}{\left(m+n\right)!}\)


\(\displaystyle y=\frac{\left(m+n+1\right)!}{m!\left(n+1\right)!}f(m+n+1)\Rightarrow f(m+n+1)=\frac{y\cdot m!\cdot \left(n+1\right)!}{\left(m+n+1\right)!}\)


\(\displaystyle z=\frac{\left(m+n+2\right)!}{\left(m+1\right)!\left(n+1\right)!}f(m+n+2) \Rightarrow f(m+n+2)=\frac{z\cdot (m+1)!\cdot (n+1)!}{\left(m+n+2\right)!}\)


再由 \(f(m+n+2)=f(m+n+1)+f(m+n)\),可得如下


\(\displaystyle\frac{z\cdot (m+1)!\cdot (n+1)!}{\left(m+n+2\right)!} =\frac{y\cdot m!\cdot \left(n+1\right)!}{\left(m+n+1\right)!}+\frac{x\cdot m!\cdot n!}{\left(m+n\right)!},\)


化簡後可得如下,

\(\displaystyle z=\frac{m+n+2}{m+1}y+\frac{(m+n+2)(m+n+1)}{(m+1)(n+1)}x.\)

Jacob 發表於 2010-7-8 08:41

想請問演算題第1題與第6題,請各位大大幫忙

想請問演算題第1題與第6題,請各位大大幫忙,感謝。

八神庵 發表於 2010-7-8 09:35

[quote]原帖由 [i]Jacob[/i] 於 2010-7-8 08:41 AM 發表 [url=https://math.pro/db/redirect.php?goto=findpost&pid=2390&ptid=973][img]https://math.pro/db/images/common/back.gif[/img][/url]
想請問演算題第1題與第6題,請各位大大幫忙,感謝。 [/quote]
第1題,請考慮左右兩邊除以4所得的餘數....mod4(一個整數平方除以4的餘數必為0或1其中一者)
第6題,由平均數可知其他九人成績和,由樣本變異數可發現其他九人成績平方和
          此時針對這九人成績使用柯西不等式,再將成績平方和的部份代換成變異數,最後開根號即所得

[[i] 本帖最後由 八神庵 於 2010-7-8 09:36 AM 編輯 [/i]]

Jacob 發表於 2010-7-8 15:38

感謝神庵大高手的解說

感謝神庵大高手的解說,讓小弟又學到了不少,謝謝。

idontnow90 發表於 2010-7-8 16:57

演算第3題.請教要怎麼算?
我用垂心的性質算了老半天不知怎麼的就是算不出來..@@...
先謝謝了~

八神庵 發表於 2010-7-8 20:13

[quote]原帖由 [i]idontnow90[/i] 於 2010-7-8 04:57 PM 發表 [url=https://math.pro/db/redirect.php?goto=findpost&pid=2397&ptid=973][img]https://math.pro/db/images/common/back.gif[/img][/url]
演算第3題.請教要怎麼算?
我用垂心的性質算了老半天不知怎麼的就是算不出來..@@...
先謝謝了~ [/quote]
你被垂心騙了,這題的重點在角A=60度
令AC=b,AB=c
在三角形ACF中,AF=ACcos60度=b/2,AE=ABcos60度=c/2
因為角AFP=角AEF=90度,由四邊形內角和可知角FPE=120度
因此
三角形ACF,三角形ABE,三角形BFP與三角形CEP均為30-60-90之三角形
由邊長比再搭配畢氏定理可以算出c與FP....這樣面積就解決了也順便把第二小題的條件也都求出來了....

idontnow90 發表於 2010-7-8 23:46

謝謝八神庵大大~終於搞定這題了~~謝謝你~

idontnow90 發表於 2010-7-9 11:08

想請教第7題...為什麼-------則對任意實數 [i][font=Times New Roman][size=4]t[/size][/font][/i],[i][font=Times New Roman][size=4]f[/size][/font][/i]([i]t[/i])[img]https://math.pro/jsMath/fonts/cmmi10/alpha/120/char3E.png[/img]0 恒成立。
\(det(A) \neq 0\)  will imply det(A) > 0 ???
另外請教第五題
謝謝~

Ellipse 發表於 2010-7-9 12:57

[quote]原帖由 [i]weiye[/i] 於 2010-6-22 02:32 PM 發表 [url=https://math.pro/db/redirect.php?goto=findpost&pid=2265&ptid=973][img]https://math.pro/db/images/common/back.gif[/img][/url]
演算題第 4 題:

245

如圖,在座標平面上,將橢圓 \(\displaystyle\frac{x^2}{4}+\frac{y^2}{1}=1\) 的上半部及圓 \(x^2 + y^2 = 4\) 的下半部組合而成一封閉曲線(其中, \(A, B\) 為此橢圓與圓的交點)。

今有一光線從此 ... [/quote]
借用一下[i]weiye[/i]圖及資料,第二部分面積,如果不要解P座標的話
令PF2=t,PF1=n,已算出cos(角P F2 O)=-1/3,則sin(角P F2 O)=2(2)^0.5 /3
在三角形P F2 F1中, t+n=2*2=4 ,所以n=4-t----------------(1)
由餘弦定理得 n^2=t^2+(2(3)^0.5)^2-2*t*2(3)^0.5*(-1/3)-------------(2)
將(1)代入(2)得 (4-t)^2=t^2+12+4(3)^0.3*t/3
化簡得t=(6-(3)^0.5)/11
所以三角形P F2 F1面積=t*2*(3)^0.5*sin(P F2 F1)*/2 =(12(6)^0.5-6(2)^0.5)/33
下半部再算梯形F2 Q R F1 面積= 10(2)^0.5/3  (高=2(6)^0.5/3)
再合併面積即為答案

[[i] 本帖最後由 Ellipse 於 2010-7-9 12:59 PM 編輯 [/i]]

八神庵 發表於 2010-7-9 14:44

[quote]原帖由 [i]idontnow90[/i] 於 2010-7-9 11:08 AM 發表 [url=https://math.pro/db/redirect.php?goto=findpost&pid=2410&ptid=973][img]https://math.pro/db/images/common/back.gif[/img][/url]
想請教第7題...為什麼-------則對任意實數 t,f(t)[img]https://math.pro/jsMath/fonts/cmmi10/alpha/120/char3E.png[/img]0 恒成立。
\(det(A) \neq 0\)  will imply det(A) > 0 ???
謝謝~ ... [/quote]
已確定僅有一解,故f(t)=del(A)不為0
你把del(A)乘開之後,會得到一個t的一個一元四次多項式,其中t^4係數為正
則當t趨近於正負無限大時,f(t)均為正無限大
就有點類似開口向上的拋物線(但實際的話中間部份會有轉折)
也就是f(t)會有local minimum.....
又因為f(t)不為0....開口又向上,所以可得知f(t)>0恆成立....就是f(t)=del(A)>0恆成立
不知道這樣的解釋您意下如何??

八神庵 發表於 2010-7-9 14:57

[quote]原帖由 [i]idontnow90[/i] 於 2010-7-9 11:08 AM 發表 [url=https://math.pro/db/redirect.php?goto=findpost&pid=2410&ptid=973][img]https://math.pro/db/images/common/back.gif[/img][/url]
另外請教第五題
謝謝~ ... [/quote]
你問的是填五(因為演算五weiye大已解)
話說這種題目
紙折一折馬上就會有一些"fu"
因為三角形ABC是等腰直角三角形,且斜邊AB=6根號2
M為AB中點,所以AM=BM=CM=3根號2
沿著CM把三角形ACM折起來,並使平面ACM與平面BCM的夾角為60度
令此四面體的高為h,由三垂線定理可知h=AMsin60度
所以V=1/3乘底面三角形BCM面積乘h

idontnow90 發表於 2010-7-10 15:52

謝謝庵大...我懂了~.

kittyyaya 發表於 2010-8-26 18:26

[size=3][font=新細明體]填充[/font][font=Times New Roman]2[/font][/size]
[font=Times New Roman][size=3]25-x2[/size][/font][font=Times New Roman][size=3] >=[/size][/font][font=Times New Roman][size=3]0[/size][/font]
[size=3][font=Wingdings][font=Wingdings]Œ[/font][/font][font=新細明體]若[/font][/size][font=Times New Roman][size=3]x-1[/size][/font]>=[font=Times New Roman][size=3]0[/size][/font]
[font=Times New Roman][size=3]25-x2 > x2-2x+1得到 [/size][/font][size=3][font=Times New Roman]-3<x<4[/font][/size]
[font=Times New Roman][size=3]所以1<= [/size][/font][font=Times New Roman][size=3]x < 4[/size][/font]
[size=3][font=Wingdings][font=Wingdings][/font][/font][font=Times New Roman]x-1<0[/font][font=新細明體]且[/font][/size]根號[size=3][font=Times New Roman]([font=Times New Roman][size=3]25-x2[/size][/font][font=Times New Roman][size=3] [/size][/font]) > |x-1|得到 -3<x<1[/font][/size]
[size=3][font=Wingdings][font=Wingdings]Ž[/font][/font][font=Times New Roman] x-1<0[/font][font=新細明體]且[font=Helvetica]根號[/font][size=3][font=Times New Roman]([font=Times New Roman][size=3]25-x2[/size][/font][font=Times New Roman][size=3] [/size][/font]) < |x-1|[/font][/size][/font][/size]
[size=3][font=Wingdings][font=Wingdings]è[/font][/font][font=Times New Roman]-5<x<-3[/font][/size]
[font=Times New Roman][size=3]
[/size][/font]
[size=3][font=新細明體]根據[/font][font=Wingdings][font=Wingdings]Œ[/font][/font][font=Wingdings][font=Wingdings][/font][/font][font=Wingdings][font=Wingdings]Ž[/font][/font][font=Times New Roman]   [/font][font=新細明體]得[/font][font=Times New Roman]-5<x<4[/font][font=新細明體]且 [/font][/size][font=Times New Roman][size=3]x 不等於 [/size][/font][font=Times New Roman][size=3]-3[/size][/font]
[font=Times New Roman][size=3] [/size][/font]
[size=3][font=新細明體]請問我討論有錯嗎[/font][color=red][font=Times New Roman]? [/font][/color][font=新細明體]可是跟官方答案[/font][/size][color=red][font=Times New Roman][size=3]([/size][/font]-5 < = x < 4[font=Times New Roman][size=3])[/size][/font][/color][font=新細明體][size=3]不同[/size][/font]
[font=新細明體][size=3][/size][/font]
[font=新細明體][size=3][color=black]另外請問選擇4真的要把20位平均數算出嗎? 若是 我只找到一位 且是估計值, 請問可否由圖型解釋[/color][/size][/font]
[font=新細明體][size=3][color=#000000]謝謝[/color][/size][/font]

kittyyaya 發表於 2010-8-26 18:34

[quote]原帖由 [i]八神庵[/i] 於 2010-7-8 08:13 PM 發表 [url=https://math.pro/db/redirect.php?goto=findpost&pid=2401&ptid=973][img]https://math.pro/db/images/common/back.gif[/img][/url]

你被垂心騙了,這題的重點在角A=60度
令AC=b,AB=c
在三角形ACF中,AF=ACcos60度=b/2,AE=ABcos60度=c/2
因為角AFP=角AEF=90度,由四邊形內角和可知角FPE=120度
因此
三角形ACF,三角形ABE,三角形BFP與三角形CEP均為30-60 ... [/quote]

請問最後一句"[font=新細明體][size=12pt]由邊長比再搭配畢氏定理可以算出[/size][/font]c[font=新細明體][size=12pt]與[/size][/font]FP",邊長是那個比那個,
因為我一直算出2b=c+6而已,找不到第二個方程式,可以麻煩在多點說明嗎?謝謝

頁: [1] 2

論壇程式使用 Discuz! Archiver   © 2001-2022 Comsenz Inc.